Lagrans metode, hvordan finne maks og min?

Her kan du stille spørsmål vedrørende problemer og oppgaver i matematikk på høyskolenivå. Alle som har kunnskapen er velkommen med et svar. Men, ikke forvent at admin i matematikk.net er spesielt aktive her.

Moderatorer: Vektormannen, espen180, Aleks855, Solar Plexsus, Gustav, Nebuchadnezzar, Janhaa

Svar
Gjest

Hei,
Kunne noen forklare hvordan man ved bruk av langrensformelen finner maks og min til denne? Altså oppgave b):

https://imgur.com/a/dOqJ2
Gjest

oppgave b) er kanksje umulig å løse, eller er det feil?
Mentos
Noether
Noether
Innlegg: 35
Registrert: 09/02-2018 17:07

For å finne maksimum og minimum til [tex]f[/tex] med begrensningen [tex]g(x,y)=x^2+y^2-1=0[/tex] løser du ligningsystemet

[tex]\begin{align} \nabla f(x,y)&=\lambda \nabla g(x,y) \\ g(x,y)&=0 \end{align}[/tex]

Her er [tex]\lambda[/tex] bare en "dummy" variabel, ofte kalt Lagrangemultiplikatoren. Det kan vises at maksimum/minimum til en funksjon med en underordnet begrensning (I dette tilfelle at det skal ligge på randen gitt ved [tex]g(x,y)=0[/tex]) må tilfredsstille et slikt system av ligninger, men det er ukjent hvilken verdi [tex]\lambda[/tex] vil ta, så den brukes kun i utregningen av ligningsystemet.
Gjest

[tex]\begin{align} \nabla f(x,y)&=\lambda \nabla g(x,y) \\ g(x,y)&=0 \end{align}[/tex]

Da får jeg:
[tex]x=1[/tex] og [tex]y=\pm\frac{\sqrt{3}}{2}[/tex]
og innsatt i [tex]\: f(x,y) \:[/tex], gir meg hhv.
[tex]f(1, \frac{\sqrt{3}}{2})=- \frac{1}{2}+\frac{sqrt{1.5}[/tex]
og
[tex]f(1, - \frac{\sqrt{3}}{2})=- \frac{1}{2}-\frac{sqrt{1.5}[/tex]

Er det riktig? Jeg har aldri løst de før, derfor jeg spør.
Gjest

kan noen vise hvordan man løser den trinn for trinn?
Gjest

(0,1) makspunkt
(0,-1) min punkt
derfor f(0,1)=1 maksverdi
f(0,-1)=-1 minimum verdi

stemmer det?
Mentos
Noether
Noether
Innlegg: 35
Registrert: 09/02-2018 17:07

Hvis du sjekker så er vel ikke dette løsninger til ligningsystemet. Jeg ville løst de lineære først, altså [tex]x[/tex] og [tex]y[/tex] uttrykt ved [tex]\lambda[/tex]. Deretter finner du [tex]\lambda[/tex], som løsning på den kvadratiske (her vil du få 2 løsninger). Bruk så disse for å få alle løsningene av [tex]x[/tex] og [tex]y[/tex]. Husk at det er gradientene som skal være lik hverandre, skriver opp hele ligningsystemet så kan du sjekke om det matcher ditt.

[tex]\begin{align} 1-2x&=2\lambda x \\ 1-2y&=2\lambda y \\ x^2+y^2&=1 \end{align}[/tex]
Gjest

Da får jeg er stygt uttrykk som dette, er det meningen at det skal være slik?

[tex](\frac{1}{2\lambda +2})^2 + (\frac{1}{2\lambda +2})^2=1[/tex]
Gjest

Altså får jeg:

[tex]\lambda=\frac{\sqrt{2}-2}{2}[/tex]

[tex]\lambda=- \frac{\sqrt{2}+2}{2}[/tex]

Er det riktig hittil? Hva gjør jeg etter dette?
Gjest

Jeg får to punkter:

[tex](\frac{1}{\sqrt{2}}, \frac{1}{\sqrt{2}})[/tex]

[tex](- \frac{1}{\sqrt{2}}, - \frac{1}{\sqrt{2}})[/tex]

Stemmer det hittil?

Er det kun to? Eller flere? Hvordan vet jeg om det er flere?

Og hva gjør jeg videre? Skal jeg sette:

[tex]f(x,y)=\lambda g(x,y)[/tex], der x og g er punktene over?
Gjest

jeg mente, at dersom jeg setter inn i den opprinnelige f (x,y) funksjonen min, så får jeg som svar at:

Toppunkt: [tex](\frac{1}{\sqrt2}, \frac{1}{\sqrt2})[/tex] med maks verdi:
[tex]f(\frac{1}{\sqrt2}, \frac{1}{\sqrt2})=\frac{2}{\sqrt2}[/tex]

Og bunn punkt [tex](\frac{-1}{\sqrt2}, \frac{-1}{\sqrt2})[/tex] med min.-verdi:
[tex]f(\frac{-1}{\sqrt2}, \frac{-1}{\sqrt2})=\frac{-2}{\sqrt2}[/tex]

Er det riktig?
Mentos
Noether
Noether
Innlegg: 35
Registrert: 09/02-2018 17:07

Gjest skrev:jeg mente, at dersom jeg setter inn i den opprinnelige f (x,y) funksjonen min, så får jeg som svar at:

Toppunkt: [tex](\frac{1}{\sqrt2}, \frac{1}{\sqrt2})[/tex] med maks verdi:
[tex]f(\frac{1}{\sqrt2}, \frac{1}{\sqrt2})=\frac{2}{\sqrt2}[/tex]

Og bunn punkt [tex](\frac{-1}{\sqrt2}, \frac{-1}{\sqrt2})[/tex] med min.-verdi:
[tex]f(\frac{-1}{\sqrt2}, \frac{-1}{\sqrt2})=\frac{-2}{\sqrt2}[/tex]

Er det riktig?
Ser bra ut.
Svar